\documentclass[11pt]{article} \usepackage{personal_commands} \usepackage[italian]{babel} \title{\textbf{Note del corso di Analisi Matematica 1}} \author{Gabriel Antonio Videtta} \date{4 aprile 2023} \begin{document} \maketitle \wip \begin{center} \Large \textbf{Teoria sulle derivate} \end{center} \begin{definition} (derivata) Sia $f : X \subseteq \RR \to \RR$. Si definisce allora \textbf{derivata} di $f$ in $\xbar \in X$ punto di accumulazione, se esiste, il seguente limite: \[Df(\xbar) = f'(\xbar) = \lim_{h \to 0} \frac{f(\xbar + h) - f(\xbar)}{h} = \lim_{x \to \xbar} \frac{f(x) - f(\xbar)}{x - \xbar}.\] \vskip 0.05in Qualora tale limite non esista, si dirà che non esiste la derivata di $f$ in $\xbar$. Si definisce anche $f' : D \subseteq X \to \RRbar$ come la funzione derivata, la quale associa ogni punto $\xbar$ in cui la derivata di $f$ esiste al valore del limite computato in $\xbar$. \end{definition} \begin{definition} $\xbar \in X$ si dice \textbf{derivabile} se e solo se esiste la derivata di $f$ in $\xbar$ e $f'(\xbar)$ è finito. \end{definition} \begin{remark}\nl \li L'insieme $D$ può essere vuoto. \\ \li Si definisce $f^{(n)}(\xbar)$ come la derivata $n$-esima di $f$ in $\xbar$. \\ \li Si definisce per convenzione $f^{(0)}(x) = f(x)$. \\ \li L'operazione di derivata è un operatore lineare. \\ \end{remark} \begin{definition} (derivata destra e sinistra) Dato $\xbar$ punto di accumulazione destro di $X$, si definisce allora \textbf{derivata destra} di $f$ in $\xbar \in X$, se esiste, il seguente limite: \[D_+ f(\xbar) = f_+'(\xbar) = \lim_{h \to 0^+} \frac{f(\xbar + h) - f(\xbar)}{h} = \lim_{x \to \xbar^+} \frac{f(x) - f(\xbar)}{x - \xbar}.\] \vskip 0.05in Qualora tale limite non esista, si dirà che non esiste la derivata destra di $f$ in $\xbar$. Analogamente, per un punto di accumulazione sinistro $\xbar \in X$, si definisce la \textbf{derivata sinistra} di $f$ in $\xbar \in X$, se esiste, il seguente limite: \[D_- f(\xbar) = f_-'(\xbar) = \lim_{h \to 0^-} \frac{f(\xbar + h) - f(\xbar)}{h} = \lim_{x \to \xbar^-} \frac{f(x) - f(\xbar)}{x - \xbar}.\] \end{definition} \begin{remark}\nl \li Se esistono sia la derivata sinistra che destra di $f$ in $\xbar$ e coincidono, allora la derivata di $f$ in $\xbar$ esiste e coincide con il valore di entrambe le due derivate. \\ \li Vale anche il viceversa, se $\xbar$ è un punto di accumulazione sia destro che sinistro: se esiste la derivata di $f$ in $\xbar$, allora sia la derivata sinistra che destra esistono e coincidono con la derivata. \end{remark} \begin{definition} Si dice che $f : X \to \RR$ è derivabile se è derivabile $\forall x \in X$. \end{definition} \begin{definition} Si dice che $f \in \cc^1$ se è derivabile e la sua funzione derivata è continua. In generale, si dice che $f \in \cc^n$ se è derivabile $n$ volte e ogni sua derivata, fino alla $n$-esima, è continua. Si pone $f \in \cc^\infty$ se $f$ è derivabile per un numero arbitrario di volte e ogni sua derivata è continua. \end{definition} \begin{proposition} Sia $f : X \to \RR$ e sia $\xbar \in X$ un punto di accumulazione di $X$. Allora: \begin{enumerate}[(i)] \item $f$ derivabile in $\xbar$ $\implies$ $f(\xbar + h) = f(\xbar) + f'(\xbar) h + o(h)$. \item Se esiste $a$ tale che $f(\xbar + h) = f(\xbar) + ah + o(h)$, allora $f$ è derivabile in $\xbar$ e $f'(\xbar) = a$. \end{enumerate} \end{proposition} \begin{proof} Se $f$ è derivabile in $\xbar$, allora $\lim_{h \to 0} \frac{f(\xbar + h) - f(\xbar) - f'(\xbar) h}{h} = \lim_{h \to 0} \frac{f(\xbar + h) - f(\xbar)}{h} - f'(\xbar) = f'(\xbar) - f'(\xbar) = 0$, da cui la prima tesi. \\ Inoltre, se esiste $a$ come nelle ipotesi, $\lim_{h \to 0} \frac{f(\xbar + h) - f(\xbar)}{h} =\lim_{h \to 0} \frac{ah + o(h)}{h} = a + \lim_{h \to 0} \frac{o(h)}{h} = a + 0 = a$, quindi $f$ è derivabile in $\xbar$ e $f'(\xbar) = a$. \end{proof} \begin{corollary} Se $f$ è derivabile in $\xbar$, allora $f$ è anche continua in $\xbar$. \end{corollary} \begin{proof} Infatti, poiché $f(x) = f(\xbar) + f'(\xbar) (x - \xbar) + o(x-\xbar)$, $\lim_{x \to \xbar} f(x) = \lim_{x \to \xbar} f(\xbar) + \lim_{x \to \xbar} f'(\xbar)(x-\xbar) + \lim_{x \to \xbar} o(x - \xbar) = \lim_{x \to \xbar} f(\xbar) = f(\xbar)$, e quindi $f$ è continua in $\xbar$. \end{proof} %TODO: trovare esempio di derivabilità infinita e non continuità \begin{proposition} Siano $f_1$, $f_2 : X \to \RR$ entrambe derivabili in $\xbar$. Allora: \begin{enumerate}[(i)] \item $(f_1 + f_2)'(\xbar) = f_1'(\xbar) + f_2'(\xbar)$, \item $(f_1f_2)'(\xbar)= f_1(\xbar) f_2'(\xbar) + f_1'(\xbar) f_2(\xbar)$. \end{enumerate} \end{proposition} \begin{proof}Poiché $f_1$ ed $f_2$ sono derivabili in $\xbar$, vale che: \[ f_1(\xbar + h) = f_1(\xbar) + f_1'(\xbar) h + o(h), \qquad f_2(\xbar + h) = f_2(\xbar) + f_2'(\xbar) h + o(h). \] \begin{enumerate}[(i)] \item $(f_1 + f_2)(\xbar + h) = (f_1 + f_2)(\xbar) + (f_1' + f_2')(\xbar) h + o(h)$. Quindi, per la proposizione precedente, $(f_1 + f_2)'(\xbar) = (f_1' + f_2')(\xbar) = f_1'(\xbar) + f_2'(\xbar)$. \item $(f_1 f_2)(\xbar + h) = (f_1 f_2)(\xbar) + (f_1(\xbar)f_2'(\xbar) + f_1'(\xbar) f_2(\xbar)) h + \underbrace{(f_1(\xbar) + f_2(\xbar)) o(h) + (f_1'f_2')(\xbar) h^2 + (f_1'(\xbar) + f_2'(\xbar))h \cdot o(h) + o^2(h))}_{=o(h)} = (f_1 f_2)(\xbar) + (f_1(\xbar)f_2'(\xbar) + f_1'(\xbar) f_2(\xbar)) h + o(h)$. Quindi, per la proposizione precedente, $(f_1 f_2)'(\xbar) = f_1(\xbar)f_2'(\xbar) + f_1'(\xbar) f_2(\xbar)$. \end{enumerate} \end{proof} \begin{proposition} Siano $f : X \to Y$ e $g : Y \to \RR$, con $f$ derivabile in $\xbar$ e $g$ derivabile in $\ybar := f(\xbar)$. Allora $g \circ f$ è derivabile in $\xbar$ e $(g \circ f)'(\xbar) = f'(\xbar) g'(\ybar)$. \end{proposition} \begin{proof} Poiché $f'(\xbar)$ è finito, $f(\xbar + h) = \ybar + f'(\xbar) h + o(h)$. Analogamente, $g(\ybar + h) = g(\ybar) + g'(\ybar) h + o(h)$. Allora $g(f(\xbar + h)) = g(\ybar + (f'(\xbar) h + o(h))) = g(\ybar) + g'(\ybar) (f'(\xbar) h + o(h)) + o(f'(\xbar) h + o(h)) = g(\ybar) + g'(\ybar) f'(\xbar) h + o(h) + o(f'(\xbar) h + o(h))$. \\ Si osserva che $\lim_{h \to 0} \frac{o(f'(\xbar) h + o(h))}{h} = \lim_{h \to 0} \frac{o(f'(\xbar) h + o(h))}{f'(\xbar) h + o(h)} \frac{f'(\xbar) h + o(h)}{h} = \lim_{h \to 0} \frac{o(f'(\xbar) h + o(h))}{f'(\xbar) h + o(h)} \lim_{h \to 0} \frac{f'(\xbar) h + o(h)}{h} = 0 \cdot f'(\xbar) = 0$, e quindi che $o(f'(\xbar) h + o(h)) = o(h)$. Allora $g(f(\xbar + h)) = g(\ybar) + g'(\ybar) f'(\xbar) h + o(h)$, da cui si conclude che $(g \circ f)'(\xbar) = g'(\ybar) f'(\xbar)$. \end{proof} \begin{proposition} Sia $f : X \to Y$ con inversa $g : Y \to X$. Sia $f$ derivabile in $\xbar$ con $f'(\xbar) \neq 0$. Sia $g$ continua in $\ybar = f(\xbar)$. Allora: \begin{enumerate}[(i)] \item $\ybar$ è un punto di accumulazione di $Y$, \item $g$ è derivabile in $\ybar$, \item $g'(\ybar) = \frac{1}{f'(\xbar)}$. \end{enumerate} \end{proposition} \begin{proof}\nl \begin{enumerate}[(i)] \item Poichè $f$ è derivabile in $\xbar$, $f$ è continua in $\xbar$. Quindi per ogni intorno $I$ di $\ybar$, esiste un intorno $J$ di $\xbar$ tale per cui $f(I \cap X \setminus \{ \xbar \}) \subseteq J$. Inoltre, $I \cap X \setminus \{\xbar\}$ non è mai vuoto, dacché, essendo $f$ derivabile in $\xbar$, $\xbar$ è un punto di accumulazione di $X$. Quindi $J$ contiene in particolare un immagine di $f$ in esso, e quindi un punto di $Y$; inoltre, tale punto è diverso da $\ybar$ dal momento che $f$ è iniettiva, essendo bigettiva. Quindi $\ybar$ è un punto di accumulazione. \item e \!(iii) Poiché $f$ è derivabile in $g(\ybar)$, $\ybar + h = f(g(\ybar + h)) = f(g(\ybar) + (\underbrace{g(\ybar + h) - g(\ybar)}_k)) = \ybar + f'(\xbar) k + o(k)$, ossia vale che: \[ h = f'(\xbar) k + o(k). \] Dal momento che $g$ è continua in $\ybar$, $k \tends{h \to 0} 0$, e quindi $o(k) \tends{h \to 0} 0$. Quindi, per $h \to 0$, $k \sim \frac{h}{f'(\xbar)}$. Si conclude dunque che $\lim_{h \to 0} \frac{g(\ybar + h) - g(\ybar)}{h} = \lim_{h \to 0} \frac{k}{h} = \frac{1}{f'(\xbar)}$. \end{enumerate} \end{proof} \begin{example} La continuità è necessaria nelle scorse ipotesi. Si può costruire infatti una funzione del tipo: \[ f(x) = \system{x & \se x \geq 0, \\ -(x+2) & \se -2 < x \leq -1.} \] dove $f'(0) = 1$, $f$ è invertibile, ma la derivata di $g$ in $0$ non esiste ($D_+ g(0) = 1$, ma $D_- g(0) = +\infty$). \end{example} \begin{theorem} (di Fermat) Sia $I$ intervallo, $f : I \to \RR$, $\xbar$ interno a $I$ punto di massimo o minimo locale con $f$ derivabile in $\xbar$, allora $f'(\xbar) = 0$. \end{theorem} \begin{example} Dimostrare che la derivata sinistra è negativa, e che quella destra è positiva nei casi che hai capito. \end{example} \begin{theorem} (di Rolle) Sia $I = [a, b] \subset \RR$ e sia $f : I \to \RR$ tale che $f$ sia continua su $I$, che $f(a) = f(b)$ e che $f$ sia derivabile in $[a, b]$. Allora $\exists \xbar \in (a, b)$ tale che $f'(\xbar) = 0$. \end{theorem} \begin{proof} Per il teorema di Weierstrass $f$ ammette un punto di massimo $M$ e uno di minimo $m$ in $I$. Se $f(a) = M$ e $f(b) = m$ o viceversa, la funzione $f$ è costante in $I$, e quindi per ogni punto in $(a, b)$ la derivata è nulla, dacché $f$ è sempre derivabile. Altrimenti, sicuramente uno tra il punto di massimo e quello di minimo appartiene a $(a, b)$. Senza perdita di generalità, si assuma che $\exists x_M \in (a, b)$ tale che $f(x_M) = M$: per il teorema di Fermat $f'(x_M) = 0$. Analogamente per il caso in cui $\exists x_m \in (a, b)$ tale che $f(x_m) = m$, da cui la tesi. \end{proof} \begin{theorem} (di Cauchy) Sia $I = [a, b] \subset \RR$ e siano $f$, $g: I \to \RR$ continue su $I$ e derivabili in $(a, b)$, con $g'$ non nulla in $(a, b)$ e $g(a) \neq g(b)$. Allora $\exists \xbar \in (a, b)$ tale che $\frac{f'(\xbar)}{g'(\xbar)} = \frac{f(b) - f(a)}{g(b)-g(a)}$. \end{theorem} \begin{proof} Si consideri la funzione $h : I \to \RR$ tale che $h(x) = f(x) - \left(\frac{f(b) - f(a)}{g(b) - g(a)} (g(x) - g(a)) + f(a)\right)$. Si osserva che $h$, essendo una somma di funzioni continue su $I$ e derivabili in $(a, b)$, è anch'essa continua su $I$ e derivabile in $(a, b)$. Inoltre $h(a) = h(b) = 0$. Quindi, per il teorema di Rolle, $\exists \xbar \in (a, b) \mid h'(\xbar) = 0 \implies \frac{f'(\xbar)}{g'(\xbar)} = \frac{f(b)-f(a)}{g(b)-g(a)}$, da cui la tesi. \end{proof} \begin{theorem} (di Lagrange) Sia $I = [a, b] \subset \RR$ e sia $f: I \to \RR$ tale che $f$ sia continua su $I$ e che $f$ sia derivabile in $(a, b)$. Allora $\exists \xbar \in (a, b)$ tale che $f'(\xbar) = \frac{f(b) - f(a)}{b-a}$, ossia la cui retta tangente è parallela alla secante che passa per $(a, f(a))$ e $(b, f(b))$. \end{theorem} \begin{proof} Si consideri $g(x) = x$, $g$ è continua in $[a, b]$ e derivabile in $(a, b)$, con derivata sempre non nulla in tale intervallo. Allora, per il teorema di Cauchy, $\exists \xbar \in (a, b) \mid f'(\xbar) = \frac{f(b)-f(a)}{b-a}$, da cui la tesi. \end{proof} \begin{proposition} Sia $I = [a, b] \subset \RR$ e sia $f : I \to \RR$ tale che $f$ sia continua su $I$ e che $f$ sia derivabile in $(a, b)$, con derivata non negativa. Allora $f$ è crescente in $[a, b]$. Analogamente, se la derivata è non positiva, $f$ è decrescente. \end{proposition} \begin{proof} Senza perdita di generalità si dimostra il caso in cui la derivata di $f$ in $(a, b)$ è non negativa (altrimenti è sufficiente considerare $g = -f$). Si considerino $c < d \in I$. Allora, per il teorema di Lagrange, $\exists \xbar \in (c, d) \mid f'(c) = \frac{f(d) - f(c)}{d-c} \implies f(d) - f(c) = \underbrace{f'(c) (d-c)}_{\geq 0} \implies f(d) \geq f(c)$, ossia che $f$ è crescente in $I$. \end{proof} \begin{remark}\nl \li L'interpretazione geometrica del teorema di Cauchy, rispetto a quella di Lagrange, è leggermente più complicata. Si consideri la curva $\gamma : \RR \to \RR^2$ tale che $\gamma(t) =(g(t), f(t))$. Si osserva che il coefficiente della retta tangente in $\xbar$ per $\gamma$ è dato da $\lim_{h \to 0} \frac{f(\xbar + h) - f(\xbar)}{g(\xbar + h) - g(\xbar)}$, che, sotto le ipotesi del teorema di Cauchy, può essere riscritto come $\frac{f'(\xbar)}{g'(\xbar)}$. Allora, il teorema di Cauchy asserisce che esiste un punto della curva $\gamma$ tale per cui la retta tangente alla curva in quel punto è parallela alla secante passante per $(g(a), f(a))$ e $(g(b), f(b))$. \end{remark} \begin{exercise} Dare un esempio di una funzione $f : \RR \to \RR$ crescente e discontinua $\forall x \in \ZZ$. \end{exercise} \begin{solution} Si consideri $f(x) = \lfloor x \rfloor$. \end{solution} \begin{exercise} Si descriva un insieme $X$ tale che i suoi punti di accumulazione sono $\{\pm 1\}$. \end{exercise} \begin{solution} Si consideri $X = \{1 + \frac{1}{n}\} \cup \{-1 + \frac{1}{n}\}$. \end{solution} \begin{exercise} Sia $f : X \to \RRbar$ continua in $\xbar$ e sia $a < f(\xbar)$. Allora esiste $J$ intorno di $\xbar$ tale che $a < f(x)$ $\forall x \in J$. \end{exercise} \begin{exercise} Sia $X \subseteq \RRbar$ e sia $\xbar$ punto di accumulazione di $X$, $f_1$, $f_2 : X \to \RRbar$. Allora: \begin{enumerate}[(i)] \item Se $f_1 \tendsto{\xbar} +\infty$ e $f_2$ è limitata inferiormente in un intorno $J$ di $\xbar$, allora $f_1(x) + f_2(x) \tendsto{\xbar} +\infty$. \item Se $f_1 \tendsto{\xbar} 0$ e $f_2$ è limitata in un intorno di $\xbar$, allora $f_1 f_2(x) \tendsto{\xbar} 0$. \item Se $f_1 \tendsto{\xbar} +\infty$ è limitata inferiormente da una costante positiva $m$ in un intorno $J$ di $\xbar$, allora $f_1 f_2 \tendsto{\xbar} +\infty$. \end{enumerate} \end{exercise} \begin{exercise} Sia $f: \RR \to \RR$ tale che: \[ f(x) = \system{x + 2x^2 \sin\left(\frac{1}{x}\right) & \se x \neq 0, \\ 0 & \altrimenti.} \] \vskip 0.05in Mostrare che $f$ è continua, che $f'(0) = 1$ e che $f'$ non è continua in zero. \end{exercise} \end{document}